Diễn Đàn MathScopeDiễn Đàn MathScope
  Diễn Đàn MathScope
Ghi Danh Hỏi/Ðáp Thành Viên Social Groups Lịch Ðánh Dấu Ðã Ðọc

Go Back   Diễn Đàn MathScope > Sơ Cấp > Việt Nam và IMO > 2015

News & Announcements

Ngoài một số quy định đã được nêu trong phần Quy định của Ghi Danh , mọi người tranh thủ bỏ ra 5 phút để đọc thêm một số Quy định sau để khỏi bị treo nick ở MathScope nhé !

* Nội quy MathScope.Org

* Một số quy định chung !

* Quy định về việc viết bài trong diễn đàn MathScope

* Nếu bạn muốn gia nhập đội ngũ BQT thì vui lòng tham gia tại đây

* Những câu hỏi thường gặp

* Về việc viết bài trong Box Đại học và Sau đại học


Trả lời Gởi Ðề Tài Mới
 
Ðiều Chỉnh Xếp Bài
Old 08-01-2015, 04:09 PM   #31
trunghy1997
+Thành Viên+
 
Tham gia ngày: Jan 2015
Bài gởi: 1
Thanks: 0
Thanked 0 Times in 0 Posts
Trích:
Nguyên văn bởi huynhcongbang View Post
Dưới đây là bài 1.

a) Với $a=0$, ta có $\left\{ \begin{align}
& {{u}_{1}}=3, \\
& {{u}_{n+1}}=\frac{1}{2}{{u}_{n}}+\frac{1}{4}\sqrt{ u_{n}^{2}+3},n\ge 1 \\
\end{align} \right.$.
Dễ thấy ${{u}_{n}}>0$ với mọi $n.$
Xét hàm số $f(x)=\frac{x}{2}+\frac{1}{4}\sqrt{{{x}^{2}}+3},x> 0$ thì ${f}'(x)=\frac{1}{2}+\frac{x}{4\sqrt{{{x}^{2}}+3}} >0$ nên $f(x)$ là hàm số đồng biến.
Ngoài ra, ${{u}_{2}}=\frac{3}{2}+\frac{\sqrt{{{3}^{2}}+3}}{4 }=\frac{3+\sqrt{3}}{2}<3={{u}_{1}}$ nên bằng quy nạp, ta chứng minh được dãy số này giảm.
Mặt khác, dãy bị chặn dưới bởi 0 nên có giới hạn hữu hạn.
Đặt $L=\lim {{u}_{n}}\ge 0$ thì $L=\frac{1}{2}L+\frac{1}{4}\sqrt{{{L}^{2}}+3} \Leftrightarrow 2L=\sqrt{{{L}^{2}}+3} \Leftrightarrow L=1$.
Vậy giới hạn cần tìm là 1.

b) Với $0\le a\le 1$, xét dãy số ${{x}_{n}},{{y}_{n}}$ lần lượt xác định bởi

$ \left\{ \begin{aligned}
& {{x}_{1}}=3, \\
& {{x}_{n+1}}=\frac{1}{2}{{x}_{n}}+\frac{1}{4}\sqrt{ x_{n}^{2}+3},n\ge 1 \\
\end{aligned} \right. $ và $\left\{ \begin{aligned}
& {{y}_{1}}=3, \\
& {{y}_{n+1}}=\frac{1}{2}{{y}_{n}}+\frac{{{n}^{2}}}{ 4{{n}^{2}}+1}\sqrt{y_{n}^{2}+3},n\ge 1 \\
\end{aligned} \right. $

Do $0\le a\le 1$ nên bằng quy nạp, dễ thấy rằng ${{x}_{n}}\ge {{u}_{n}}\ge {{y}_{n}}$ với mọi $n\ge 1$.
Theo câu $a$, ta đã chứng minh được $\lim {{x}_{n}}=1$.
Ta sẽ chứng minh bằng quy nạp rằng ${{y}_{n}}\ge 1-\frac{2}{n}$ với mọi $n\ge 2$. (*)
Thật vậy, với $n=2$, dễ dàng thấy rằng (*) đúng.

Giả sử $(*)$ đúng với $n$ thì ta có ${{y}_{n}}\ge 1-\frac{2}{n}\ge 0$. Suy ra $$ \dfrac{1}{2}{{y}_{n}}+ \dfrac{{{n}^{2}}}{4{{n}^{2}}+1} \sqrt{y_{n}^{2}+3} \ge \dfrac{1}{2}\left( 1-\frac{2}{n} \right)+ \dfrac{{{n}^{2}}}{4{{n}^{2}}+1} \sqrt{{{\left( 1-\frac{2}{n} \right)}^{2}}+3} . $$ Ta cần chứng minh:

$\dfrac{1}{2} \left( 1-\dfrac{2}{n} \right)+ \dfrac{{{n}^{2}}}{4{{n}^{2}}+1} \sqrt{{{ \left( 1-\dfrac{2}{n} \right)}^{2}}+3} \ge 1-\dfrac{2}{n+1}$
$\Leftrightarrow \dfrac{{{n}^{2}}}{4{{n}^{2}}+1} \sqrt{{{ \left( 1-\dfrac{2}{n} \right)}^{2}}+3} \ge \left( 1-\dfrac{2}{n+1} \right)-\dfrac{1}{2} \left( 1-\dfrac{2}{n} \right) $
$ \Leftrightarrow \dfrac{2n}{4{{n}^{2}}+1} \sqrt{{{n}^{2}}-n+1}\ge \dfrac{{{n}^{2}}-n+2}{2n(n+1)} $
$ \Leftrightarrow 4{{n}^{4}}{{(n+1)}^{2}}({{n}^{2}}-n+1) \ge {{(4{{n}^{2}}+1)}^{2}}{{({{n}^{2}}-n+2)}^{2}}$
$\Leftrightarrow 16{{n}^{4}}{{(n+1)}^{2}}({{n}^{2}}-n+1) \ge {{(4{{n}^{2}}+1)}^{2}}{{({{n}^{2}}-n+2)}^{2}} $

Ta thấy bất đẳng thức trên đúng với $n=1,2,3$, ta xét $n \ge 4$.

Chú ý rằng

$16{{n}^{2}}{{(n+1)}^{2}}\ge {{(4{{n}^{2}}+1)}^{2}} \Leftrightarrow 4n(n+1)\ge 4{{n}^{2}}+1$ đúng và
${{n}^{2}}({{n}^{2}}-n+1)\ge {{({{n}^{2}}-n+2)}^{2}}\Leftrightarrow n{{(n-2)}^{2}}\ge 4$ với mọi $n\ge 4$.

Từ đó ta được ${{x}_{n}}\ge {{u}_{n}}\ge 1-\frac{2}{n}$ đúng với mọi $n$, mà $\lim {{x}_{n}}=\lim \left( 1-\frac{2}{n} \right)=1$ nên ta có $\lim {{u}_{n}}=1$.
em làm gần giống tn. Làm tn ra được quy nạp Yn đấy ạ
[RIGHT][I][B]Nguồn: MathScope.ORG[/B][/I][/RIGHT]
 
trunghy1997 is offline   Trả Lời Với Trích Dẫn
Old 08-01-2015, 04:16 PM   #32
Infinitedream1
+Thành Viên+
 
Tham gia ngày: Nov 2014
Bài gởi: 6
Thanks: 13
Thanked 1 Time in 1 Post
Không kẻ thêm đường phụ biến đổi lượng giác có được không ạ...
[RIGHT][I][B]Nguồn: MathScope.ORG[/B][/I][/RIGHT]
 
Infinitedream1 is offline   Trả Lời Với Trích Dẫn
Old 08-01-2015, 04:18 PM   #33
quocbaoct10
+Thành Viên Danh Dự+
 
quocbaoct10's Avatar
 
Tham gia ngày: Oct 2012
Đến từ: THPT chuyên Lê Quý Đôn-Nha Trang-Khánh Hòa
Bài gởi: 539
Thanks: 292
Thanked 365 Times in 217 Posts
Trích:
Nguyên văn bởi analysis90 View Post
Em có thể giải thích tại sao không? nó chỉ đơn thuần là bài toán về đa thức và số phức. Bài tổ hợp này thực chất là bài tổ hợp mà trường PTNK đã từng cho thi và cũng là bài tổ hợp của Romani năm 2003.
Ý em là đi thi thì không dùng được hàm sinh do nó là toán cao cấp. Còn bài của PTNK thì nó hoàn toàn có thể giải bằng truy hồi đơn giản bởi 2 dãy số, không cần thiết phải dùng tới số phức ạ .

Phân tích một chút về bài tổ: bài thi HSG năm nay thì khác một chút ở điều kiện, $\{2,0,1,5 \}$ thì nó có 1 số chia hết 3, 1 số chia 3 dư 1 và 2 số chia 3 dư 2, khi lập song ánh để tạo quan hệ truy hồi thì ta có dãy $b_n$ (số các số không chia hết cho 3) phải tách ra thành 2 dãy chia 3 dư 1 (gọi là dãy x, số các số trong x là $x_n$) và dư 2 (tương tự, gọi y và $y_n$), do khi xét một số thuộc dãy $a_{n+1}$, bỏ đi số 2 tận cùng thì được 1 số thuộc dãy chia 3 dư 1, bỏ đi số 5 thì cũng được 1 dãy chia 3 dư 1, tức là từ $a_{n+1}$ có thể có được $2x_n$, khi bỏ đi số tận cùng là số 1 thì chỉ được 1 số thuộc dãy y, hay chỉ được $y_n$, mà $2x_n+y_n \neq b_n$. còn bài PTNK hình như có đk là các số có n chữ số tạo từ $\{3,5,7,9 \}$, đk có 2 số chia hết cho 3, 1 số chia 3 dư 1 và 1 số chia 3 dư 2. Làm tương tự trên thì ta có: $a_{n+1}=2a_n+x_n+y_n$, mà $x_n+y_n$ lại chính bằng $b_n$ nên như thế kết hợp với $a_n+b_n=4^n$, đỡ đi 1 dãy truy hồi, nên nó đơn giản hơn. Nên nếu nhìn từ điểm này thì giải bài 3 bằng phương pháp số phức sẽ đơn giản hơn rất rất nhiều.
[RIGHT][I][B]Nguồn: MathScope.ORG[/B][/I][/RIGHT]
 
__________________
i'll try my best.
quocbaoct10 is offline   Trả Lời Với Trích Dẫn
Old 08-01-2015, 04:23 PM   #34
huynhcongbang
Administrator

 
huynhcongbang's Avatar
 
Tham gia ngày: Feb 2009
Đến từ: Ho Chi Minh City
Bài gởi: 2,413
Thanks: 2,165
Thanked 4,188 Times in 1,381 Posts
Gửi tin nhắn qua Yahoo chát tới huynhcongbang
Trích:
Nguyên văn bởi tohoproirac View Post
Song song rồi sao nữa thầy ????
Cụ thể là thế này đây bạn:

Bài 4a.


Gọi $K,L$ lần lượt là giao điểm của $(I)$ với $BC$.
Ta có $AK\cdot AF=AL\cdot AE \Rightarrow \frac{AK}{AL}=\frac{AE}{AF}=\frac{AB}{AC} \Rightarrow KL\parallel BC$.

Do $(I)$ tiếp xúc với $BC$ tại $D$ nên
\[\frac{B{{D}^{2}}}{C{{D}^{2}}}=\frac{BF\cdot BK}{CE\cdot CL}=\frac{BF}{CE}\cdot \frac{BK}{CL}=\frac{BF}{CE}\cdot \frac{AB}{AC}=\frac{BF}{CE}\cdot \frac{BE}{CF}=\frac{\cot B}{\cot C}. \]
Vậy $\dfrac{BD}{CD}=\sqrt{\dfrac{\cot B}{\cot C}}$.
[RIGHT][I][B]Nguồn: MathScope.ORG[/B][/I][/RIGHT]
 
Hình Kèm Theo
Kiểu File : png Capture.PNG (15.4 KB, 918 lần tải)
__________________
Sự im lặng của bầy mèo
huynhcongbang is offline   Trả Lời Với Trích Dẫn
The Following User Says Thank You to huynhcongbang For This Useful Post:
babysama (08-01-2015)
Old 08-01-2015, 05:28 PM   #35
pco
+Thành Viên+
 
pco's Avatar
 
Tham gia ngày: Dec 2011
Bài gởi: 528
Thanks: 560
Thanked 195 Times in 124 Posts
Bài 4. Nếu $(I)$ tiếp xúc với $(O)$ tại $D$ thì ta cũng có $\frac{DB}{DC}= \sqrt{\frac{\cot B}{\cot C}}$.
[RIGHT][I][B]Nguồn: MathScope.ORG[/B][/I][/RIGHT]
 
__________________
"People's dreams... will never end!" - Marshall D. Teach.
pco is offline   Trả Lời Với Trích Dẫn
The Following User Says Thank You to pco For This Useful Post:
thaygiaocht (08-01-2015)
Old 08-01-2015, 06:22 PM   #36
Lucifer1998
+Thành Viên+
 
Tham gia ngày: Nov 2014
Bài gởi: 19
Thanks: 3
Thanked 4 Times in 4 Posts
Trích:
Nguyên văn bởi Fool's theorem View Post
Bài 3: (Cách giải sử dụng số phức)
Xét đa thức $P(x)=(1+x+x^2+x^5)^K=\sum_{n_1,..,n_K \in (2,0,1,5)}x^{n_1+...+n_K}$
Dễ thấy số số chia hết cho $3$, không vượt quá $10^K$ mà lập từ các chữ số ${2,0,1,5}$ là tổng các hệ số dạng $a_{3m}$
Ta có $\sum a_{3m} = \frac{P(1)+P(e)+P(e^2)}{3}$ với $e= cos\frac{2\pi}{3} + isin\frac{2\pi}{3}$, $e^3=1$ và $1+e+e^2=0$.
$ S=\frac{P(1)+P(e)+P(e^2)}{3}=\frac{4^K+e^{2K}+e^K} {3}$
Nếu $K$ chia hết cho 3 thì $S=\frac{4^K+2}{3}$
Nếu $K$ đồng dư 1 mod 3 thì $S=\frac{4^K+e+e^2}{3} = \frac{4^K-1}{3}$
Nếu $K$ đồng dư 2 mod 3 thì $S=\frac{4^K-1}{3}$
Anh ơi nếu bài giải chỉ lập luận đơn giản thế này liệu có được điểm tối đa ko ạ? Em hoang mang quá
[RIGHT][I][B]Nguồn: MathScope.ORG[/B][/I][/RIGHT]
 
Lucifer1998 is offline   Trả Lời Với Trích Dẫn
Old 08-01-2015, 08:38 PM   #37
Khoa Bảo
+Thành Viên+
 
Tham gia ngày: Jan 2015
Bài gởi: 1
Thanks: 0
Thanked 1 Time in 1 Post
Bài 3 mình làm ra đáp án ntn đúng với mọi k lớn hơn hoặc bằng 1
$n=\frac{4^k-1}{3}+\left \lfloor \frac{k}{3} \right \rfloor$
Làm trong bài chưa kịp rút gọn
[RIGHT][I][B]Nguồn: MathScope.ORG[/B][/I][/RIGHT]
 
Khoa Bảo is offline   Trả Lời Với Trích Dẫn
The Following User Says Thank You to Khoa Bảo For This Useful Post:
thaygiaocht (08-01-2015)
Old 08-01-2015, 08:41 PM   #38
kien10a1
+Thành Viên+
 
kien10a1's Avatar
 
Tham gia ngày: Feb 2011
Đến từ: Vĩnh Yên- Vĩnh Phúc
Bài gởi: 371
Thanks: 43
Thanked 263 Times in 153 Posts
Gửi tin nhắn qua Yahoo chát tới kien10a1
Cách khác cho bài 1b, mình vẫn thấy khá phức tạp.
Ta chỉ cần chứng minh rằng dãy với a=1 hội tụ về 1 là xong.
+, Giả sử mọi số hạng trong dãy đều lớn hơn 1, khi đó ta sử dụng đánh giá $u_n^2+3<4u_n^2 $, và có dãy giảm.
+, Nếu trong dãy có $u_{N}<1 $ thì dễ quy nạp từ N trở đi cả dãy đều gồm số nhỏ hơn 1. Trong trường hợp này ta lại xét 2 trường hợp nhỏ:
-, Nếu có n mà n>N và $u_{n+1}>u_n $ thì từ đây quy nạp được dãy tăng, và suy ra dãy hội tụ.
-, Ngược lại thì dãy phải giảm từ N, bị chặn dưới bởi 0 nên cũng hội tụ, thay vào được giới hạn và loại TH này.
[RIGHT][I][B]Nguồn: MathScope.ORG[/B][/I][/RIGHT]
 
__________________
Quay về với nơi bắt đầu

thay đổi nội dung bởi: kien10a1, 08-01-2015 lúc 08:56 PM
kien10a1 is offline   Trả Lời Với Trích Dẫn
The Following 2 Users Say Thank You to kien10a1 For This Useful Post:
huynhcongbang (08-01-2015), thaygiaocht (08-01-2015)
Old 09-01-2015, 01:33 AM   #39
huynhcongbang
Administrator

 
huynhcongbang's Avatar
 
Tham gia ngày: Feb 2009
Đến từ: Ho Chi Minh City
Bài gởi: 2,413
Thanks: 2,165
Thanked 4,188 Times in 1,381 Posts
Gửi tin nhắn qua Yahoo chát tới huynhcongbang
Trích:
Nguyên văn bởi trunghy1997 View Post
em làm gần giống tn. Làm tn ra được quy nạp Yn đấy ạ
À thì cứ làm theo các nhu cầu phát sinh thôi em. Cần có $\lim y_n = 1$, mà quy nạp trực tiếp không được (bị ngược dấu), nên "hạ giá" nó xuống. Thay vì lớn hơn 1 thì cho lớn hơn 1 trừ cho cái gì đó có liên quan đến $n$ mà giới hạn là 0 thôi. Dễ nhất sẽ là dạng $\dfrac{k}{n}$. Ban đầu anh thử $k=1$ thì thấy hơi khó chứng minh, đến $k=2$ thì biến đổi trơn tru hơn nên làm, thế thôi.
------------------------------
Trích:
Nguyên văn bởi Lucifer1998 View Post
Anh ơi nếu bài giải chỉ lập luận đơn giản thế này liệu có được điểm tối đa ko ạ? Em hoang mang quá
Trong bài đó có đoạn tổng các hệ số $a_{3m}$ bằng $\dfrac{P(1)+P(e)+P(e^2)}{3}$ có lẽ nên chứng minh lại, nếu không thì theo anh sẽ bị trừ mất 1-2 điểm (về cơ bản thì nó chính là định lý RUF). Các lập luận còn lại thì sáng sủa rồi.
------------------------------
Trích:
Nguyên văn bởi Infinitedream1 View Post
Không kẻ thêm đường phụ biến đổi lượng giác có được không ạ...
Vẫn OK mà em. Cách biến đổi lượng giác, không thông qua giao điểm kia tất nhiên là tự nhiên hơn rồi.
[RIGHT][I][B]Nguồn: MathScope.ORG[/B][/I][/RIGHT]
 
__________________
Sự im lặng của bầy mèo

thay đổi nội dung bởi: huynhcongbang, 09-01-2015 lúc 01:41 AM Lý do: Tự động gộp bài
huynhcongbang is offline   Trả Lời Với Trích Dẫn
The Following 3 Users Say Thank You to huynhcongbang For This Useful Post:
DenisO (09-01-2015), Infinitedream1 (11-01-2015), thaygiaocht (09-01-2015)
Old 09-01-2015, 11:41 AM   #40
huynhcongbang
Administrator

 
huynhcongbang's Avatar
 
Tham gia ngày: Feb 2009
Đến từ: Ho Chi Minh City
Bài gởi: 2,413
Thanks: 2,165
Thanked 4,188 Times in 1,381 Posts
Gửi tin nhắn qua Yahoo chát tới huynhcongbang
Đây là đề ngày 2, cũng lấy từ facebook của bạn Trung, đội tuyển Long An.


[RIGHT][I][B]Nguồn: MathScope.ORG[/B][/I][/RIGHT]
 
Hình Kèm Theo
Kiểu File : jpg vmo day 2.jpg (104.2 KB, 671 lần tải)
__________________
Sự im lặng của bầy mèo
huynhcongbang is offline   Trả Lời Với Trích Dẫn
The Following 4 Users Say Thank You to huynhcongbang For This Useful Post:
9nho10mong (09-01-2015), HoangHungChels (09-01-2015), son235 (09-01-2015), thaygiaocht (09-01-2015)
Old 09-01-2015, 11:59 AM   #41
haojack123
+Thành Viên+
 
Tham gia ngày: Jun 2013
Bài gởi: 7
Thanks: 6
Thanked 23 Times in 6 Posts
Đặt $Q(x)={{x}^{3}}-{{x}^{2}}+x=x\left( {{x}^{2}}-x+1 \right),$ ta có:
$\begin{aligned}
& {{f}_{n}}(x)=3x{{f}_{n-1}}(x)+\left( 1-x-2{{x}^{2}} \right){{f}_{n-1}}(x) \\
& \Leftrightarrow {{f}_{n}}(x)-\left( x+1 \right){{f}_{n-1}}(x)=\left( 2x-1 \right){{f}_{n-1}}(x)-\left( 2x-1 \right)\left( x+1 \right){{f}_{n-2}}(x) \\
& \Leftrightarrow {{f}_{n}}(x)-\left( x+1 \right){{f}_{n-1}}(x)=\left( 2x-1 \right)\left[ {{f}_{n-1}}(x)-\left( x+1 \right){{f}_{n-2}}\left( x \right) \right] \\
& \\
\end{aligned}$
Suy ra
$\begin{aligned}
& {{f}_{n}}(x)-\left( x+1 \right){{f}_{n-1}}(x)={{\left( 2x-1 \right)}^{n-1}}\left[ {{f}_{1}}(x)-\left( x+1 \right){{f}_{0}}(x) \right]={{\left( 2x-1 \right)}^{n-1}}\left( x-2 \right) \\
& \Leftrightarrow {{f}_{n}}(x)-{{\left( 2x-1 \right)}^{n}}=\left( x+1 \right)\left[ {{f}_{n-1}}(x)-{{\left( 2x-1 \right)}^{n-1}} \right] \\
\end{aligned}$
Từ đây ta có: ${{f}_{n}}(x)-{{\left( 2x-1 \right)}^{n}}={{\left( x+1 \right)}^{n}}\left[ {{f}_{0}}(x)-{{\left( 2x-1 \right)}^{0}} \right]={{\left( x+1 \right)}^{n}}$ hay:
${{f}_{n}}(x)={{\left( 2x-1 \right)}^{n}}+{{\left( x+1 \right)}^{n}}$
${{f}_{n}}(x)\vdots Q(x)\Rightarrow {{f}_{n}}\left( 0 \right)=0\Leftrightarrow {{\left( -1 \right)}^{n}}+1=0\Leftrightarrow n$ lẽ.
Lại có: $Q\left( -2 \right)=-14,\,\,\,{{f}_{n}}\left( -2 \right)=-\left( {{5}^{n}}+1 \right).$ Suy ra ${{5}^{n}}+1\vdots 7.$
Xét $n=3k+1,$ vì $n$ lẽ nên $k$ chẵn suy ra ${{5}^{3k+1}}+1\equiv 6\left( \bmod 7 \right).$
Xét $n=3k+2,$ vì $n$ lẽ nên $k$ lẽ suy ra ${{5}^{3k+2}}+1\equiv 4\left( \bmod 7 \right).$
Xét $n=3k,$ vì $n$ lẽ nên $k$ lẽ suy ra ${{5}^{3k}}+1\equiv 0\left( \bmod 7 \right).$
Vậy $n=3k$ với $k$ là số tự nhiên lẽ.
Ta có: ${{f}_{n}}(x)={{f}_{3k}}(x)={{\left( 2x-1 \right)}^{3k}}+{{\left( x+1 \right)}^{3k}}=\left[ {{\left( 2x-1 \right)}^{3}}+{{\left( x+1 \right)}^{3}} \right]M=9x\left( {{x}^{2}}-x+1 \right)M.$
Với $M={{\left( 2x-1 \right)}^{3\left( k-1 \right)}}-{{\left( 2x-1 \right)}^{3\left( k-2 \right)}}{{\left( x+1 \right)}^{3}}+...-...+...-{{\left( 2x-1 \right)}^{3}}{{\left( x+1 \right)}^{3\left( k-2 \right)}}+{{\left( x+1 \right)}^{3\left( k-1 \right)}}.$
Từ đây suy ra ${{f}_{n}}(x)={{f}_{3k}}(x)\vdots Q(x).$
Vậy $n=3k$ với $k$ là số tự nhiên lẽ là tất cả các giá trị cần tìm.
[RIGHT][I][B]Nguồn: MathScope.ORG[/B][/I][/RIGHT]
 

thay đổi nội dung bởi: haojack123, 09-01-2015 lúc 10:34 PM
haojack123 is offline   Trả Lời Với Trích Dẫn
The Following 11 Users Say Thank You to haojack123 For This Useful Post:
9nho10mong (09-01-2015), babysama (09-01-2015), baotram (09-01-2015), BlackSelena (09-01-2015), CTK9 (09-01-2015), huynhcongbang (09-01-2015), Juliel (09-01-2015), nguyentatthu (09-01-2015), pco (09-01-2015), son235 (09-01-2015), thaygiaocht (09-01-2015)
Old 09-01-2015, 12:08 PM   #42
thaygiaocht
+Thành Viên+
 
thaygiaocht's Avatar
 
Tham gia ngày: Aug 2012
Đến từ: Chuyên Hà Tĩnh
Bài gởi: 165
Thanks: 793
Thanked 216 Times in 93 Posts
Trích:
Nguyên văn bởi huynhcongbang View Post
Đây là đề ngày 2, cũng lấy từ facebook của bạn Trung, đội tuyển Long An.

Câu 5. Làm thủ công tính đến $f_0,f_1,f_2,f_3,f_4$ sẽ phát hiện số hạng đầu $3,5,9,17$ dự đoán quy luật là $2^n+1, $ cụ thể:
1. $f_0=2;$
2. $f_1=3x;$
3. $f_2=5x^2-2x+2;$
4. $f_3=9x^3-9x^2+9x;$
5. $f_4=17x^4-37x^3+30x^2-4x+2.$
Dùng phương pháp đoán, ta nghi ngờ $f_n=(2x+-1)^n+(x+-1)^n.$
Kiểm tra kỹ thấy $f_n=(2x-1)^n+(x+1)^n.$
Chứng minh điều này bằng quy nạp rất đơn giản.
Phần sau tương đương $f_n(0)=0$ và $f_n(k)=f_n(k^2)=0$ với $k \ne 1$ là căn bậc 3 của đơn vị, $k^3=1.$
Thay vào tính toán đại số ta thu được đáp số.
[RIGHT][I][B]Nguồn: MathScope.ORG[/B][/I][/RIGHT]
 
__________________
https://www.facebook.com/thaygiaocht

thay đổi nội dung bởi: thaygiaocht, 09-01-2015 lúc 04:11 PM
thaygiaocht is offline   Trả Lời Với Trích Dẫn
The Following 6 Users Say Thank You to thaygiaocht For This Useful Post:
babysama (09-01-2015), caubemetoan96 (09-01-2015), HoangHungChels (09-01-2015), Infinitedream1 (09-01-2015), Juliel (09-01-2015), vantienducdh (09-01-2015)
Old 09-01-2015, 12:18 PM   #43
huynhcongbang
Administrator

 
huynhcongbang's Avatar
 
Tham gia ngày: Feb 2009
Đến từ: Ho Chi Minh City
Bài gởi: 2,413
Thanks: 2,165
Thanked 4,188 Times in 1,381 Posts
Gửi tin nhắn qua Yahoo chát tới huynhcongbang
Bài 6 chính là một dạng tương tự của bài 3 IMO 1983:

http://www.artofproblemsolving.com/F...b46215#p366618

Cho $a,b,c$ là các số nguyên dương có $(a,b,c)=1$. Chứng minh rằng $2abc-ab-bc-ca$ là số nguyên lớn nhất không biểu diễn được dưới dạng $xab+ybc+zca$.

Chú ý rằng phương trình đã cho viết lại là: $x.a^2+y.(6a)+z.(6^2)=n$.
[RIGHT][I][B]Nguồn: MathScope.ORG[/B][/I][/RIGHT]
 
__________________
Sự im lặng của bầy mèo
huynhcongbang is offline   Trả Lời Với Trích Dẫn
The Following User Says Thank You to huynhcongbang For This Useful Post:
thaygiaocht (09-01-2015)
Old 09-01-2015, 12:24 PM   #44
Hennmarsk
+Thành Viên+
 
Tham gia ngày: Dec 2014
Bài gởi: 6
Thanks: 4
Thanked 3 Times in 3 Posts
Cho mình hỏi đáp án bài 6.a có phải a=1 hay là a=1; a=5 hả mọi người?
[RIGHT][I][B]Nguồn: MathScope.ORG[/B][/I][/RIGHT]
 
Hennmarsk is offline   Trả Lời Với Trích Dẫn
The Following User Says Thank You to Hennmarsk For This Useful Post:
dangvip123tb (09-01-2015)
Old 09-01-2015, 12:33 PM   #45
vantienducdh
+Thành Viên+
 
Tham gia ngày: Aug 2014
Đến từ: 12 Toán THPT chuyên LQĐ-Quảng Trị
Bài gởi: 45
Thanks: 35
Thanked 11 Times in 10 Posts
Năm ni nghe nói điểm sàn giải sẽ tăng,không biết có đúng thế không
[RIGHT][I][B]Nguồn: MathScope.ORG[/B][/I][/RIGHT]
 
__________________
MỘT BÀI TOÁN HAY LÀ BÀI TOÁN KHÔNG ÁP DỤNG NHIỀU KỸ THUẬT MÀ BÀI TOÁN ĐÓ PHẢI ĐẾN TỰ NHIÊN,DỄ HIỂU NHẤT
vantienducdh is offline   Trả Lời Với Trích Dẫn
Trả lời Gởi Ðề Tài Mới

Bookmarks

Ðiều Chỉnh
Xếp Bài

Quuyền Hạn Của Bạn
You may not post new threads
You may not post replies
You may not post attachments
You may not edit your posts

BB code is Mở
Smilies đang Mở
[IMG] đang Mở
HTML đang Tắt

Chuyển đến


Múi giờ GMT. Hiện tại là 03:47 PM.


Powered by: vBulletin Copyright ©2000-2024, Jelsoft Enterprises Ltd.
Inactive Reminders By mathscope.org
[page compression: 114.08 k/130.62 k (12.66%)]